Difference between revisions of "2008 AMC 12A Problems/Problem 2"

(typo)
m (typo)
Line 8: Line 8:
  
 
==See Also==
 
==See Also==
{{AMC12 box|year=2008|ab=A|num-b=1|num-a=3}}}
+
{{AMC12 box|year=2008|ab=A|num-b=1|num-a=3}}

Revision as of 15:43, 17 February 2008

Problem

What is the reciprocal of $\frac{1}{2}+\frac{2}{3}$?

$\textbf{(A)} \frac{6}{7} \qquad \textbf{(B)} \frac{7}{6}  \qquad \textbf{(C)} \frac{5}{3}  \qquad \textbf{(D)}  3  \qquad \textbf{(E)}  \frac{7}{2}$

Solution

$\left(\frac{1}{2}+\frac{2}{3}\right)^{-1}=\left(\frac{3}{6}+\frac{4}{6}\right)^{-1}=\left(\frac{7}{6}\right)^{-1}=\frac{6}{7}\Rightarrow A$.

See Also

2008 AMC 12A (ProblemsAnswer KeyResources)
Preceded by
Problem 1
Followed by
Problem 3
1 2 3 4 5 6 7 8 9 10 11 12 13 14 15 16 17 18 19 20 21 22 23 24 25
All AMC 12 Problems and Solutions